Prueba matemática rigurosa del principio de incertidumbre a partir de los primeros principios [duplicado]

Mientras buscaba una explicación intuitiva para el Principio de Incertidumbre de Heisenberg (pregunta relacionada a continuación), se mencionó un enfoque axiomático para establecer el principio de incertidumbre. ¿Podría alguien señalar una fuente con pasos detallados y explicaciones de los primeros principios?

Pregunta relacionada

¿Se puede explicar intuitivamente el Principio de Incertidumbre de Heisenberg?

Algunas (re)búsquedas revelarán la siguiente prueba (y otras) que quizás no sean captadas de inmediato por personas que no están familiarizadas con cierta terminología/conceptos.

Prueba relacionada

Principio de incertidumbre de Heisenberg prueba científica

En la respuesta anterior, no está claro

  1. Cómo el producto de vectores, PAG q , se descompone en partes reales e imaginarias?

  2. ¿Cómo el valor esperado de PAG q al cuadrado es el cuadrado de las partes real e imaginaria por separado?

  3. ¿Cómo ambas cosas cuadradas son positivas ya que se trata de una porción compleja?

  4. ¿Qué tan cuadradas son las cosas positivas significa que el lado izquierdo es más grande que un cuarto del cuadrado del conmutador?

  5. ¿Por qué el conmutador no cambia por el cambio? [ PAG , q ] = [ pag , X ] = ?

Tenga en cuenta que yo era un estudiante decente de física (tal vez no, pero todavía estoy muy interesado) que se desvió hacia las ciencias sociales para realizar estudios de posgrado. Por lo tanto, estoy un poco oxidado con la notación y la terminología. Cualquier sugerencia para repasar los conceptos y llenar los vacíos en las explicaciones existentes sería muy apreciada. Entiendo que mis preguntas pueden parecer muy triviales u obvias para los expertos, por lo tanto, disculpe mi ignorancia de los conceptos básicos.

EDITAR: tenga en cuenta que esta no es una pregunta duplicada. La pregunta a la que se hace referencia para marcar esta pregunta como duplicada está vinculada a esta pregunta. La presente pregunta se originó a raíz de algunas dudas respecto a la otra pregunta. Eliminaré esta edición una vez que esté claro.

Las pruebas axiomáticas pertenecen a las matemáticas. La necesidad del HUP como principio proviene de los datos: la necesidad de modelar la naturaleza ondulatoria de las probabilidades para mediciones en el microcosmos. Todo lo que las pruebas matemáticas pueden decirle es: que el modelo matemático se ajusta a los datos con éxito.
Repito: El HUP se llama principio pero se podría haber llamado "ley", como en las "leyes gravitatorias", o "regla" como en "Born rule". En física, "principios", "leyes", "postulados" son palabras que se usan para enfatizar que se deben imponer restricciones adicionales en las matemáticas y las soluciones de las matemáticas, para seleccionar el subconjunto de soluciones que SE ADAPTA A LOS DATOS Y OBSERVACIONES. No hay forma de "probar datos". Por eso las teorías físicas se validan y no se prueban. La consistencia de las predicciones matemáticas con los datos valida la teoría física. no lo prueba
Por lo tanto, las respuestas que obtiene muestran que el HUP surge de las matemáticas utilizadas, no es una prueba del HUP. Es una prueba de la consistencia del modelo matemático con los datos. Cuando algo se demuestra correctamente en matemáticas, no puede cambiar. Los postulados/principios/leyes físicos, etc. están sujetos a cambios para adaptarse a las observaciones. Ejemplo: para altas velocidades, los postulados de Galileo se descartan y se obtienen postulados de la relatividad especial que se ajustan a los datos.
@annav Gracias por tus aclaraciones. Entiendo lo que dices. Mi edición se basa en que Qmechanic marcó esta pregunta como duplicada, lo cual no es así, ya que hace referencia y busca aclaraciones sobre la pregunta en función de la cual la marcó como duplicada.

Respuestas (2)

La prueba aquí muestra que los operadores hermitianos A , B satisfacer σ A σ B 1 2 | [ A , B ] | . (Es una consecuencia de la desigualdad de Cauchy-Schwarz en el espacio de Hilbert). Dado que [ X , pag ] = i , σ X σ pag 2 .

Gracias por su tiempo para señalar esta otra respuesta. He visto la otra prueba que mencionas. No tengo claro cómo se descompone el producto y la parte posterior. He editado la pregunta para reflejar esto. ¿Podría por favor elaborar esos pasos?

1) Es un producto de operadores. Y no se descomponen tanto en partes reales e imaginarias, sino en partes autoadjuntas y antiautoadjuntas.

Si tomamos autoadjunto A , B operadores lineales en algún espacio de Hilbert adecuado, está claro que

A B = 1 2 ( A B + B A ) + 1 2 ( A B B A ) ,
desde
1 2 ( A B + B A ) + 1 2 ( A B B A ) = 1 2 A B + 1 2 B A + 1 2 A B 1 2 B A = 2 1 2 A B = A B .
Ahora, desde A y B son auto-adjuntos,
( A B + B A ) = B A + A B = B A + A B = A B + B A ,
entonces este "anticonmutador", A B + B A , es autoadjunto si A y B son.

Sin embargo, si nos fijamos en el conmutador, A B B A ,

( A B B A ) = B A A B = B A A B = ( A B B A ) ,
el conmutador de operadores autoadjuntos A , B es anti-auto-adoint.

Ahora, la razón por la que los llamó "reales" e "imaginarios" es porque en el espacio de todos los operadores lineales de un espacio vectorial unitario, los operadores autoadjuntos son análogos a los números reales dentro del campo de números complejos, y los operadores antiautoadjuntos son análogos a los números imaginarios.

2, y el resto) Primero debemos notar que tomamos valores esperados con respecto a los estados cuánticos. Si nuestra partícula está en el estado | ψ , entonces el valor esperado de A con respecto al estado | ψ es A ψ = ψ | A | ψ , de donde podemos ver que el valor esperado es lineal.

Ahora bien,

A B = 1 2 ( A B + B A ) + 1 2 ( A B B A ) = 1 2 A B + B A + 1 2 A B B A .

Debemos tener en cuenta que el valor esperado de un operador está relacionado con sus valores propios. El valor esperado de un operador autoadjunto es real, porque sus valores propios son reales, y el valor esperado de un operador antiautoadjunto es imaginario, porque los valores propios son imaginarios. Además, debido a que el conmutador [ A , B ] = A B B A es antiautoadjunto, existe un operador autoadjunto C , para cual [ A , B ] = i C , desde i C es entonces antiauto-adjunto ( C es autoadjunto, pero el i signo de permuta).

Tenga en cuenta ahora que la publicación que estaba citando estaba mal en el sentido de que no tomamos el cuadrado del valor esperado, sino el cuadrado del valor absoluto del valor esperado.

Pero desde A B B A = [ A , B ] = i C = i C , y luego tomamos el valor absoluto al cuadrado de A B :

| A B | 2 = 1 4 A B + B A 2 + 1 4 C 2 ,
pero entonces
| A B | 2 1 4 C 2 ,
y todo aquí es menos que ( Δ A ) 2 ( Δ B ) 2 , Lo que significa que
( Δ A ) ( Δ B ) 1 2 C ,
pero para X y pag , C = i [ X , pag ] = i i I = I , entonces 1 2 C = / 2 .

Tenga en cuenta que la prueba que vinculó es ligeramente incorrecta, según mi primera vista, o usó algunas manipulaciones algebraicas implícitas que no encuentro triviales, pero la línea general de pensamiento es la misma.

Gracias por esta respuesta detallada. ¿Podría proporcionar una referencia independiente (lo más concisa posible) para familiarizarse con la notación, la terminología y los conceptos discutidos aquí? Muy agradecido por su ayuda.
@user249613 Aprendí QM de las notas de clase publicadas por el profesor que me impartió el curso de QM. Está disponible gratuitamente, pero solo en húngaro, así que dudo que te sirva de algo. La mayor parte de esa conferencia se ha basado en el libro de QM de JJ Sakurai "Modern Quantum Mechanics", aunque es un gran libro. También puedo recomendar el libro QM de Griffiths, que es un libro introductorio muy bueno (el de Sakurai es más avanzado), sin embargo, Griffiths usa funciones de onda en notación de función la mayor parte del tiempo en lugar del enfoque de notación de freno de vector de estado más abstracto.
Muy agradecido por sus pacientes aclaraciones. La suposición clave para derivar el principio de incertidumbre parece ser la relación entre los operadores conjugados canónicos; X y pag , [ X , pag ] = i . He visto en este enlace ( en.wikipedia.org/wiki/Canonical_commutation_relation ) que esto significa que los dos operadores son transformadas de Fourier entre sí. ¿Podría señalar una buena fuente para obtener más intuición y pasos detallados sobre estos operadores conjugados canónicos, sus transformadas de Fourier y por qué necesitan mantener esta relación de conmutador?
@Qmechanic Tenga en cuenta que esto no es un duplicado. Esta pregunta menciona explícitamente y busca aclaraciones sobre partes de la publicación que usted señala como la que tiene la respuesta.